3. What is the solution to this equation?
m/4 - 3 = 6 + m/2

Answers

Answer 1

Answer:

m = -36

Step-by-step explanation:

Solve for m:

m/4 - 3 = m/2 + 6

Hint: | Put the fractions in m/4 - 3 over a common denominator.

Put each term in m/4 - 3 over the common denominator 4: m/4 - 3 = m/4 - 12/4:

m/4 - 12/4 = m/2 + 6

Hint: | Combine m/4 - 12/4 into a single fraction.

m/4 - 12/4 = (m - 12)/4:

(m - 12)/4 = m/2 + 6

Hint: | Put the fractions in m/2 + 6 over a common denominator.

Put each term in m/2 + 6 over the common denominator 2: m/2 + 6 = m/2 + 12/2:

(m - 12)/4 = m/2 + 12/2

Hint: | Combine m/2 + 12/2 into a single fraction.

m/2 + 12/2 = (m + 12)/2:

(m - 12)/4 = (m + 12)/2

Hint: | Make (m - 12)/4 = (m + 12)/2 simpler by multiplying both sides by a constant.

Multiply both sides by 4:

(4 (m - 12))/4 = (4 (m + 12))/2

Hint: | Cancel common terms in the numerator and denominator of (4 (m - 12))/4.

(4 (m - 12))/4 = 4/4×(m - 12) = m - 12:

m - 12 = (4 (m + 12))/2

Hint: | In (4 (m + 12))/2, divide 4 in the numerator by 2 in the denominator.

4/2 = (2×2)/2 = 2:

m - 12 = 2 (m + 12)

Hint: | Write the linear polynomial on the left hand side in standard form.

Expand out terms of the right hand side:

m - 12 = 2 m + 24

Hint: | Move terms with m to the left hand side.

Subtract 2 m from both sides:

(m - 2 m) - 12 = (2 m - 2 m) + 24

Hint: | Combine like terms in m - 2 m.

m - 2 m = -m:

-m - 12 = (2 m - 2 m) + 24

Hint: | Look for the difference of two identical terms.

2 m - 2 m = 0:

-m - 12 = 24

Hint: | Isolate terms with m to the left hand side.

Add 12 to both sides:

(12 - 12) - m = 12 + 24

Hint: | Look for the difference of two identical terms.

12 - 12 = 0:

-m = 24 + 12

Hint: | Evaluate 24 + 12.

24 + 12 = 36:

-m = 36

Hint: | Multiply both sides by a constant to simplify the equation.

Multiply both sides of -m = 36 by -1:

(-m)/(-1) = -36

Hint: | Any nonzero number divided by itself is one.

(-1)/(-1) = 1:

Answer: m = -36


Related Questions

Bobby bought the following items at the school store: 10 pencils for $0.21 each, 8 pens for $0.45 each, and 2 posters for $0.55 each. How much money did Bobby spend in all?

Group of answer choices
$5.70
$1.21
$4.70
$6.80

Answers

The answer is $1.21
0.21 + 0.45 + 0.55 = 1.21

Determine the y-intercept of the
equation below. Write your answer as
an ordered pair.
5x + 8y = 40

Answers

The answer would be (0,5)

Solve: Sin(11 pi/2 + x) = -1/2 for [Pi, 2Pi].
StartFraction 4 pi Over 3 EndFraction
StartFraction 5 pi Over 3 EndFraction
StartFraction 7 pi Over 6 EndFraction
StartFraction 11 pi Over 6 EndFraction

Answers

Answer: B

The answer above is correct. It would be 5pi/3.

Step-by-step explanation: Edge2021

I just got it right on my quiz.

The value of the x is equal to the 5pi/3.

The expression is

[tex]\sin \left(11\cdot \frac{\pi }{2}+x\right)=-\frac{1}{2},\:\pi \le \:x\le \:2\pi[/tex]

The value of the given expression.

What is the expression?

An expression or mathematical expression is a finite combination of symbols that is well-formed according to rules that depend on the context.

We have,

[tex]\sin \left(11\cdot \frac{\pi }{2}+x\right)=-\frac{1}{2},\:\pi \le \:x\le \:2\pi[/tex]

[tex]11\cdot \frac{\pi }{2}+x=\frac{7\pi }{6}+2\pi n,\:11\cdot \frac{\pi }{2}+x=\frac{11\pi }{6}+2\pi n[/tex]

[tex]x=2\pi n-\frac{13\pi }{3},\:x=2\pi n-\frac{11\pi }{3}[/tex]

[tex]x=\frac{5\pi }{3}[/tex]

Therefore, The value of the x is equal to the 5pi/3.

To learn more about the sin function visit:

https://brainly.com/question/16818112

#SPJ2

Select the correct answer from each drop-down menu. Lisa specializes in baking lemon cupcakes. She bakes 3 dozen cupcakes every hour. The cost (in dollars) of making n cupcakes is given by the function C(n) = 60 + 0.45n. The function that models the number of cupcakes Lisa makes in h hours is n(h) = . The cost function in terms of hours, h, is given by . Lisa's cost for making cupcakes for 2 hours is .

Answers

Answer:

Step-by-step explanation:

from the question, we are told that Lisa bakes 3dozens cupcakes every hour, to know the amount she makes in h hours, we will use the equality postulate as shown;.

3 dozens = 1hour

n(h) = h hour

Cross multiply

n(h) × 1 = 3×h

n(h) = 3h

Hence the function that models the number of cupcakes Lisa makes in h hours is n(h) = 3h

To get the cost function in terms of hours, we will find the composite function C(n(h))

C(n(h)) = C(3h)

Given C(n) = 60 + 0.45n

C(3h) is derived by substituting n as 3h in the function as shown;

C(3h) = 60+0.45(3h)

C(3h) = 60+1.35h

Hence the cost function in terms of hours, h, is given by 60+1.35h

To get the cost for making cupcakes for 2 hours, we will substitute h = 2 into the expression C(h) = 60+1.35h

C(2) = 60+1.35(2)

C(2) = 60+2.70

C(2) = 62.70

Hence the cost for making cupcakes for 2 hours is $62.70

Answer:

The function that models the number of cupcakes Lisa makes in h hours is n(h) = 36h. The cost function in terms of hours, h, is given by 60 + 16.2h. Lisa's cost for making cupcakes for 2 hours is $92.40

.

Step-by-step explanation:

Solve h+p=7(k -4) for k.

Answers

Answer:

k= h/7 + p/7 +4

Dude help me pleaseeeeeee

Answers

Answer: -25/42

Step-by-step explanation: For multiplying fractions, it’s multiplying numerator by numerator and denominator by denominator. 5 x 5 = 25 and 6 x 7 = 42 and then add the - since it’s negative. It can not simplify

The answer is -25/42

Help pleaseI don’t understand

Answers

Answer:

Angle 3 is 65

Angle 8 is 115

Step-by-step explanation:

Angle 3 is 65
Angle 8 is 315 I think

sporting goods store is offering a 20% discount on shoes. Mariana also has a $5 off coupon that can be applied to her
purchase. She is planning to buy a pair of shoes that originally costs $89. Will the final price be lower if the discount is applied before the
coupon or if the coupon is applied before the discount? Justify your response

Answers

The discount price will be the same either way. (The commutative property)

20 percent of $89.00= 17.80$

$89.00 - ($5.00 x $17.80) = $66.20

$89.00 - ($17.80 x $5.00) = $66.20

The final price be lower if the discount is applied before the coupon.

What is the fundamental principle of multiplication?

Multiplication is the mathematical operation that is used to determine the product of two or more numbers. If an event can occur in m different ways and if following it, a second event can occur in n different ways, then the two events in succession can occur in m × n different ways.

Given that sporting goods store is offering a 20% discount on shoes. Mariana also has a $5 off coupon that can be applied to her

purchase.

She is planning to buy a pair of shoes that originally costs =  $89.

The discount price is the same either way.

Thus 20 percent of $89.00= 17.80$

Therefore, we have;

$89.00 - ($5.00 x $17.80) = $66.20

$89.00 - ($17.80 x $5.00) = $66.20

Learn more about multiplications;

https://brainly.com/question/14059007

#SPJ2

Question 1 Please. I need help.

Answers

Answer:

74.22

Step-by-step explanation:

You are supposed to add 52.39+21.83 because of part whole postulate (PWP) So, now you get 74.22, which is your answer.

74.22 is the correct answer to this question

Write a fraction that is equivalent to 3/4
PLEASE I WILL GIVE YOU BRAINLEIST

Answers

Answer:

6 / 8

Step-by-step explanation:

Just multiply by 2

Answer:

3/4=6/8=9/12=12/16 and so on

Step-by-step explanation:

Just use the multiples of 3/4

Pls mark me brainliest!!!

A fraction bar labeled 1. Under the 1 are 2 boxes containing one-half. Under the 2 boxes are 4 boxes containing one-fourth. 2 one-fourths are circled.
A fraction bar labeled one-fourth. Under the one-fourth are 2 boxes containing one-half. Under the 2 boxes are 4 boxes containing 1. 2 boxes containing 1 are circled.
A fraction bar labeled 1. Under the 1 are 4 boxes containing one-fourth. Under the 4 boxes are 8 boxes containing one-half. One box containing one-half is circled.
A fraction bar labeled 1. Under the 1 are 4 boxes containing one-fourth. Under the 4 boxes are 2 boxes containing one-half. One box containing one-half is circled.

Answers

Answer:

i think option D

Step-by-step explanation:

Answer:the man above me

Step-by-step explanation:

Gina's family goes to a theme park. The family pays a $12 fee to park their car and buys admission tickets for all 5 family members. Her family pays a total of $132 . Which equation can be used to solve for t , the price of one admission ticket?
A.
5(12+t)=132
B.
5t+12=132
C.
5(12)+t=132
D.
5+12+t=132

Answers

Answer:

B

Step-by-step explanation:

5t + 12 = 132

We know the price of parking and we know what they paid in total. There are 5 members with the unknown price.

Answer:

B

Step-by-step explanation:

You're trying to find how much a ticket, which is t. you need 5 tickets for the family, plus the twelve dollar fee which is there anyways for the car, and it add up to 132. subtract 12 from 132 and you get 120. 120 divided by 5 is 24. so 24 dollars for a ticket, but the equation you would need is 5t+12=132

A weather balloon is launched in Texas from an altitude of 300 feet above sea level. If the balloon is rising at a constant speed of 80 feet per minute, write an equation that can be used to find t, the time in minutes it will take for the balloon to reach an altitude of 10,500 feet?

Answers

10500 - 300 + 80t yay

What is the value of (-5)^4?

Answers

Answer:

-20

Step-by-step explanation:

Answer:

-625

Step-by-step explanation:

-5× -5 = 25

25 × 25= 625

Modeling Task 1
A cell phone company charges a monthly rate of $14 and $0.25 a minute per call.
Judy's total cell phone bill is $50. How many minutes, m, was Judy charged for?
Make sure to define your variable, write an equation, solve the equation, and
write a sentence.

Answers

Answer:

144 minutes

Step-by-step explanation:

The equation will be 14 + 0.25x = 50

50 -  14 = 36

0.25x = 36

36/0.25 = 144 minutes

Ten is no less then a number q multiplied by 6.

Answers

Answer:

10 > 6q

Step-by-step explanation:

Assuming you are asking what this word problem looks like as an equation, then it would be:

10 > 6q

please help asap, i will give brainliest

Answers

Answer:

1) 110*,     2) 84*,       3) 80*

Step-by-step explanation:

         These are intersected parallel lines (with only 1 intersecting line), so having missing angles in the questions 1 and 2 opposite from the shown angle is giving the answer away. However, on answer 3, it is on the same side, so for that angle you will need to subtract the angle of the intersecting line by 100* to get 80*. Check by adding the two. 100* + 80* = 180*. Hope this helps.

Which number produces a rational number when added to 0.5?
A. –1.73205081…
B. 1/4
C. 0.54732871…
D. √3

Answers

Answer:

Option B is correct 1/4.

Step-by-step explanation:

To find : Which number produces a rational number when added to 0.5?

Solution :

A rational number is defined as number written in simple fraction or in p/q form.

An irrational number is defined as number which cannot be written in p/q form.

We know that,

A rational number adding a rational number gives you a rational number.

A rational number adding an irrational number gives you a irrational number.

Now we take each option and check.

Option A : -1.73205081…

Number has no end and have infinite number of digits so it is irrational number.

No, this number not produce a rational number.

Option B : 1/4=0.25

Number is in form p/q so it is rational number.

Yes, this number produce a rational number.

0.25+0.5=0.75 a rational number.

Option C : 0.54732871…

Number has no end and have infinite number of digits so it is irrational number.

No, this number not produce a rational number.

Option D : \sqrt{3}

\sqrt{3} cannot be written in ratio so it is irrational number.

No, this number not produce a rational number.


what is the value of x​

Answers

Answer:

2.5

Step-by-step explanation:

log8(32^1.5)=2.5

b^2 + 35 = −12b Solve each equation by factoring
I really need help with understanding this step by step, thank you. I am giving 40 points for this.

Answers

Answer:

[tex]b=-5, -7[/tex]

Step-by-step explanation:

We have the equation [tex]b^2+35=-12b[/tex] and we want to solve it by factoring.

First, let's move all the stuff to one side so that the other side is 0. So, let's add [tex]12b[/tex] to both sides. This yields:

[tex]b^2+12b+35=0[/tex]

Let's review how to factor. If we have an equation in the following form:

[tex]ax^2+bx+c=0[/tex]

Where a, b, and c are the coefficients of the variable (in our example, our variable is b), then we must find two numbers, p and q, such that:

[tex]pq=ac \text{ (p times q equals a times c) and}\\p+q=b \text{ (p plus q equals b)}[/tex]

Our equation can be rewritten as:

[tex](1)b^2+(12)b+(35)=0[/tex]

So, our a is 1, b is 12, and c is 35.

Therefore, we need to find two numbers that when multiplied together yields a(c) = 1(35) = 35 and we added together yields b = 12.

From here, we just have to guess and check. We can start by listing all the factors of 35. There aren't in fact that many:

[tex]35: 1\text{ and } 35, 5\text{ and } 7[/tex]

1 + 35 is 36, not 12. However, 5 + 7 is indeed 12. So, our two numbers p and q are 5 and 7.

Now what we've found our two numbers, we substitute the b term for our two numbers. We have:

[tex]b^2+12b+35=0[/tex]

We will substitute 12b for 5b + 7b:

[tex]b^2+5b+7b+35=0[/tex]

From here, we factor by grouping. From the first two terms, factor out a b:

[tex]b(b+5)+7b+35=0[/tex]

And from the last two terms, factor out a 7:

[tex]b(b+5)+7(b+5)=0[/tex]

Now, notice that both terms have a [tex](b+5)[/tex]. So, by using grouping or the reverse of the distribute property, we can write:

[tex](b+7)(b+5)=0[/tex]

Notice that if we distribute the left term into the right, we get [tex]b(b+5)+7(b+5)=0[/tex], so our equation is indeed equivalent.

So now we. have:

[tex](b+7)(b+5)=0[/tex]

We can now use the Zero Product Property to acquire:

[tex]b+7=0\text{ or } b+5=0[/tex]

Solve for b for each case. Therefore, the solutions of our equation are:

[tex]b=-7\text{ or } b=-5[/tex]

Notes:

If we can't find two numbers p and q that satisfy our conditions, this means that the equation cannot be factored. So, we will use alternative methods to solve our equation.

whats the slope intercept form of y=3x-2

Answers

Slope:3
Y intercept: (0,-2)

What is the slope of a line perpendicular to the line graphed below?

Answers

Answer:

-3

Step-by-step explanation:

Original slope: 1/3

Perpendicular slopes are the negative reciprocals of the original slope.

How many halves are in 19 wholes?

Answers

Answer:

38

Step-by-step explanation:

Hope this helps!!

38 hope it’s correct

whats the rate of change?​

Answers

Every time x is added by one, y is added by three.

Three scoops of whey powder contain 45 grams of protein. How many scoops of whey powder contain 135 grams of protein?

Please help ‼️

Answers

3

135/45=3

Please mark me as Brainliest

The answer is 405 because if you multiply 3 and 135 that would lead you to your answer.

Write the income equation for a small business that
begins with $150 and grows by 20% each year. When
will the company's income be $300?

Answers

Answer:

in 4 years

Step-by-step explanation:

because you should find the 20% of 150

150×20=3000

3000÷100= 30

so 20% of 150 is 30

then you keep adding 30 to the results till you reach 300

The equation is 150+30x=300 and it takes 3 years for the company to earn $300.

Given that, a small business begins with $150 and grows by 20% each year.

We need to find when will the company's income by $300.

What is an equation?

In mathematics, an equation is a formula that expresses the equality of two expressions, by connecting them with the equals sign =.

Now, 20% of 150=30

Let x be the number of years.

So, f(x)=150+30x

⇒150+30x=300

⇒30x=150

x=3

The equation is 150+30x=300 and it takes 3 years for the company to earn $300.

To learn more about the equations visit:

https://brainly.com/question/10413253.

#SPJ2

What is the first step in solving the equation 3.5n+6.4= 42.5?
0 Add 6.4 to each side of the equation.
O Subtract 6.4 from each side of the equation.
O Multiply each side of the equation by 3.5.
O Divide each side of the equation by 6.4.

Answers

Answer add 6.4 to each side explanation:

The second option. Subtract 6.4 from each side of the equation.

A yellowfin tuna weighs about 380 pounds. A bluefin tuna weighs 620 about pounds. If one fish of each species were caught per​ day, about how many more pounds of bluefin tuna than yellowfin tuna would be caught in a​ week? Solve the problem two different ways.

Answers

Answer:

1680 pounds

Step-by-step explanation:

Given that :

Weight of yellow fin tuna = 380 pounds

Weight of bluefin tuna = 620 pounds

If one fish of each species were caught per day

about how many more pounds of bluefin tuna than yellowfin tuna would be caught in a​ week?

Total pound of bluefin tuna caught in a week:

7 * 620 = 4340

Total pound of yellow tuna caught in a week:

7 * 380 = 2660

(4340 - 2660) = 1680 pounds

Triangle ADB is congruent to triangle ? Please HELP!!

Answers

Answer:

CBD

Step-by-step explanation:

What are 3 words that can be associated with positive numbers?

Answers

Gain,profit, increase
Other Questions
What do the arrows represent? Write an informative essay in whichyou explain your vision of a utopia.Describe three aspects of your utopiausing supporting details.Please I need help Given the function g defined by the formula g(x) =x 5/2x-5 find the following: Consider the following figure where l1 and l2 are parallel and cut by transversals g1and g2.Part A: Determine the values of M, N, V and R. Which term describes a repository that holds pairs of entries to translate a domain name to an IP address? Select 2 options.a. name serverb. domain serverc. root serverd. domain name systeme. domain lookup Who was Johannes Gutenberg and what was the significance of his invention? What would be the state of the following list after each of the first four passes in a Bubble sort, sorting into ascending sequence? (a) 65, 34, 28, 68, 52, 21 What are the major elements present in carbohydrates? C,H,OC,H,O,NC,H,O,N,PC,Fe,Si . If you apply heat to the copper bar, its density will (gene) a segment of DNA that ______ for a trait and is transferred from parent to offspring. What goods did Europeans try and buy and why did they want an all-water routeto Asia? Name an irrational number ( in radical form) located between 11 and 12. Explain answer. In "The Boy Who Invented TV," how did Philo's farm chores help him envision how a TV might work? What do you think our generation must protest Select the idea that supports how the Anasazi farmers adapt crops to grow in "the colder climate and shorter growing season," according to Selection 1. A by planting seeds from wild varieties B by storing seeds through the long winter C by importing seeds from other countries D by breeding seeds from sturdy varieties Can someone please help me here! Tell me the answer Ill give a brainliest to the one who is right! Nemecek Brothers make a single product on two separate production lines, A and B. Its labor force is equivalent to 1000 hours per week, and it has $3000 outlay weekly on operating costs. It takes 1 hour and 4 hours to produce a single item on lines A and B, respectively. The cost of producing a single item is $5 on line A and $4 on line B. (a) Write the inequality that expresses the labor information. (b) Write the inequality that expresses the cost information. Imagine youre a supervisor in a field of criminal justice and have received reports of miscommunication and misunderstandings stemming from diverse issues. You decide to write a memorandum to your staff to educate them on some communication strategies. I missed my zoom lesson today and I don't understand the material we covered. I'm not sure how to answer this question. Thank you in advance for your help Please and thank you